調和数を含んだ級数(Euler-sum)とゼータ関数 part9

前回の記事は:

調和数を含んだ級数(Euler-sum)とゼータ関数 part8

多重対数関数の定義と初歩的な関係式の知識が前提となります。こちらから:

多重対数関数(ポリログ)の関係式一覧・証明付き

Today's theme

$n=0,1,2,\cdots$ に対して調和数 $H_n$ を次のように定義する。$$H_n=1+\frac{1}{2}+\cdots+\frac{1}{n}\quad,\quad H_0=0$$このとき、\begin{eqnarray}\sum_{n=1}^\infty\frac{H_{4n}}{n^2}&=&\frac{67}{8}\zeta(3)-2\pi G\\\sum_{n=1}^\infty\frac{H_{4n}}{n^3} &=& 52\Li_4\left(\frac{1}{2}\right)-\frac{43}{80}\pi^4+\frac{91}{2}\zeta(3)\ln2-\frac{13}{6}\pi^2\ln^22+\frac{13}{6}\ln^42\\\sum_{n=1}^\infty\frac{H^2_{4n}}{n^2}&=&13\Li_4\left(\frac{1}{2}\right)+\frac{2\pi^4}{15}+\frac{91}{8}\zeta(3)\ln2-\frac{7}{24}\pi^2\ln^22\\&&+\frac{13}{24}\ln^42-4G^2+2\pi G\ln2+4\pi\mathfrak{I}\Li_3(1-i)\\\sum_{n=1}^\infty\frac{(-1)^nH_{2n}}{n^3} &=& 5\Li_4\left(\frac{1}{2}\right)-\frac{13\pi^4}{192}+\frac{35}{8}\zeta(3)\ln2-\frac{5\pi^2}{24}\ln^22+\frac{5\ln^42}{24}\end{eqnarray}\begin{eqnarray}\sum_{n=1}^\infty\frac{(-1)^nH_{2n}}{n^2} &=& \frac{23}{16}\zeta(3)-\pi G\end{eqnarray}

4倍添え字の方針

今回のは $H_{4n}$ すなわち、調和数の添え字が4倍となっています。前回の3倍添え字の方法を応用してみましょう。

数列 $a_n$ を考えます。\begin{eqnarray*}\sum_{n=1}^\infty&& a_n+\sum_{n=1}^\infty a_ni^n+\sum_{n=1}^\infty a_n(-1)^n+\sum_{n=1}^\infty a_n(-i)^n\\&&=(1+i-1-i)a_1+(1-1+1-1)a_2+(1-i-1+i)a_3+4a_4+\cdots\\&&=4\sum_{n=1}^\infty a_{4n}\end{eqnarray*}最左辺について、$a_n(-i)^n=(a_ni^n)^*$ であることも考慮して

\begin{equation}\sum_{n=1}^\infty a_{4n}=\frac{1}{4}\left(\sum_{n=1}^\infty a_n+\sum_{n=1}^\infty (-1)^na_n+2\mathfrak{R}\sum_{n=1}^\infty a_n i^n\right)\tag{1}\end{equation}

公式の導出①

(1)より$$\sum_{n=1}^\infty\frac{H_{4n}}{n^2}=4\left[\sum_{n=1}^\infty\frac{H_{n}}{n^2}+\sum_{n=1}^\infty\frac{(-1)^nH_{n}}{n^2}+2\mathfrak{R}\sum_{n=1}^\infty\frac{i^nH_{n}}{n^2}\right]$$

調和数を含んだ級数とゼータ関数 part1

で示した式$$\sum_{n=1}^\infty\frac{H_n}{n^2}=2\zeta(3)$$$$\sum_{n=1}^\infty \frac{(-1)^{n}H_n}{n^2}=-\frac{5}{8}\zeta(3)$$より\begin{equation}\sum_{n=1}^\infty\frac{H_{4n}}{n^2}=4\left[\frac{11}{8}\zeta(3)+2\mathfrak{R}\sum_{n=1}^\infty\frac{i^nH_{n}}{n^2}\right]\tag{2}\end{equation}

調和数を含んだ級数(Euler-sum)とゼータ関数 part8

で示した式

\begin{eqnarray}\sum_{n=1}^\infty\frac{H_n}{n^2}x^n &=& \Li_3(x)-\Li_3(1-x)+\ln(1-x)\Li_2(1-x)\\&&\quad+\frac{1}{2}\ln x\ln^2(1-x)+\zeta(3)\tag{3}\end{eqnarray}

より\begin{eqnarray*}\sum_{n=1}^\infty\frac{i^nH_{n}}{n^2}&=& \Li_3(i)-\Li_3(1-i)+\ln(1-i)\Li_2(1-i)\\&&\quad+\frac{1}{2}\ln i\ln^2(1-i)+\zeta(3)\\&=& \Li_3(i)-\Li_3(1-i)+\left(\frac{\ln2}{2}-\frac{\pi}{4}i\right)\Li_2(1-i)\\&&\quad+\frac{\pi}{4}i\left(\frac{\ln2}{2}-\frac{\pi}{4}i\right)^2+\zeta(3)\end{eqnarray*}

多重対数関数(ポリログ)の関係式一覧・証明付き

より\begin{equation}\Li_3(i)=-\frac{3}{32}\zeta(3)+\frac{\pi^3i}{32}\tag{4}\end{equation}

Integrals and Miscellaneous 7

の「2022/7/5B」より\begin{equation}\mathfrak{R}\Li_3(1-i)=\frac{35}{64}\zeta(3)+\frac{\pi^2}{32}\ln2\tag{5}\end{equation}同ページの「2022/7/5C」より\begin{equation}\Li_2(1-i)=\frac{\pi^2}{16}-i\left(\frac{\pi}{4}\ln2+G\right)\tag{6}\end{equation}これらを適用して\begin{equation}\sum_{n=1}^\infty\frac{i^nH_{n}}{n^2}=\frac{23}{64}\zeta(3)-\frac{\pi}{4}G-i\left(\frac{\ln2}{2}G+\frac{\pi}{16}\ln^22+\mathfrak{I}\Li_3(1-i)\right)\tag{6a}\end{equation}実部をとります。\begin{eqnarray*}\mathfrak{R}\sum_{n=1}^\infty\frac{i^nH_{n}}{n^2}=\frac{23}{64}\zeta(3)-\frac{\pi}{4}G\end{eqnarray*}(2)に放り込みます。$$\sum_{n=1}^\infty\frac{H_{4n}}{n^2}=4\left[\frac{11}{8}\zeta(3)+\frac{23}{32}\zeta(3)-\frac{\pi}{2}G\right]$$よって次の級数公式を得ます。

\begin{equation}\sum_{n=1}^\infty\frac{H_{4n}}{n^2}=\frac{67}{8}\zeta(3)-2\pi G\tag{7}\end{equation}

$\approx4.312$ です。

公式の導出②

調和数を含んだ級数とゼータ関数 part2

で示した$$\sum_{n=1}^\infty\frac{H_n}{n^3}=\frac{\pi^4}{72}$$\begin{eqnarray*}\sum_{n=1}^\infty \frac{(-1)^nH_{n}}{n^3}&&=-\frac{11\pi^4}{360}+2\Li_4\left(\frac{1}{2}\right)+\frac{7}{4}\zeta(3)\ln2\\&&\quad-\frac{\pi^2}{12}\ln^22+\frac{\ln^42}{12}\end{eqnarray*}により\begin{eqnarray}\sum_{n=1}^\infty\frac{H_{4n}}{n^3}&=&16\Bigl[2\Li_4\left(\frac{1}{2}\right)-\frac{\pi^4}{60}+\frac{7}{4}\zeta(3)\ln2-\frac{\pi^2}{12}\ln^22\\&&+\frac{\ln^42}{12}+2\mathfrak{R}\sum_{n=1}^\infty\frac{H_n}{n^3}i^n\Bigr]\tag{8}\end{eqnarray}残った級数は次の表示により求めます。\begin{eqnarray*}\sum_{n=1}^\infty\frac{H_n}{n^3}x^n&=&2\Li_4(x)+\Li_4\left(\frac{x}{x-1}\right)-\Li_4(1-x)-\ln(1-x)\Li_3(x)\\&&+\frac{1}{24}\ln^4(1-x)-\frac{1}{6}\ln x\ln^3(1-x)+\frac{\pi^2}{12}\ln^2(1-x)\\&&+\zeta(3)\ln(1-x)+\frac{\pi^4}{90}\end{eqnarray*}$x=i$ として\begin{eqnarray*}\sum_{n=1}^\infty\frac{H_n}{n^3}i^n&=&2\Li_4(i)+\Li_4\left(\frac{1}{1+i}\right)-\Li_4(1-i)-\ln(1-i)\Li_3(i)\\&&+\frac{1}{24}\ln^4(1-i)-\frac{1}{6}\ln i\ln^3(1-i)+\frac{\pi^2}{12}\ln^2(1-i)\\&&+\zeta(3)\ln(1-i)+\frac{\pi^4}{90}\\&=&2\Li_4(i)+\Li_4\left(\frac{1}{1+i}\right)-\Li_4(1-i)-\left(\frac{\ln2}{2}-\frac{\pi}{4}i\right)\Li_3(i)\\&&+\frac{1}{24}\left(\frac{\ln2}{2}-\frac{\pi}{4}i\right)^4-\frac{\pi i}{12}\left(\frac{\ln2}{2}-\frac{\pi}{4}i\right)^3+\frac{\pi^2}{12}\left(\frac{\ln2}{2}-\frac{\pi}{4}i\right)^2\\&&+\zeta(3)\left(\frac{\ln2}{2}-\frac{\pi}{4}i\right)+\frac{\pi^4}{90}\end{eqnarray*}(4)を得たのと同じ過去記事から$$\Li_4(i)=-\frac{7}{128}\zeta(4)+i\beta(4)$$さらに$$\Li_4(-z)+\Li_4\left(-\frac{1}{z}\right)=-\frac{7}{4}\zeta(4)-\frac{1}{24}\ln^4\frac{1}{z}-\frac{\pi^2}{12}\ln^2\frac{1}{z}$$で $z=\frac{-1}{1+i}$ とすることにより$$\Li_4\left(\frac{1}{1+i}\right)=-\Li_4(1+i)+\frac{1313\pi^4}{92160}-\frac{\ln^42}{384}+\frac{11\pi^2}{768}\ln^22+\frac{i\pi}{64}\ln^32+\frac{7i \pi^3}{256}\ln2$$(4)より$$\Li_3(i)=-\frac{3}{32}\zeta(3)+\frac{\pi^3i}{32}$$これらを適用して実部をとりましょう。\begin{eqnarray*}\mathfrak{R}\sum_{n=1}^\infty\frac{H_n}{n^3}i^n&=&-\frac{7\pi^4}{5760}-\mathfrak{R}\Li_4(1+i)+\frac{1313\pi^4}{92160}-\frac{\ln^42}{384}+\frac{11\pi^2}{768}\ln^22\\&&-\mathfrak{R}\Li_4(1-i)+\frac{3}{64}\zeta(3)\ln2-\frac{\pi^4}{128}\\&&+\frac{1}{24}\left(\frac{\pi^4}{256}-\frac{3\pi^2}{32}\ln^22+\frac{\ln^42}{16}\right)+\frac{\pi}{12}\left(\frac{\pi^3}{64}-\frac{3\pi}{16}\ln^22\right)\\&&+\frac{\pi^2}{12}\left(\frac{\ln^22}{4}-\frac{\pi^2}{16}\right)+\frac{\ln2}{2}\zeta(3)+\frac{\pi^4}{90}\end{eqnarray*}これをゴリゴリ計算します。$\mathfrak{R}\Li_4(1+i)+\mathfrak{R}\Li_4(1-i)=2\mathfrak{R}\Li_4(1+i)$ ですので\begin{eqnarray*}\mathfrak{R}\sum_{n=1}^\infty\frac{H_n}{n^3}i^n&=&\frac{29\pi^4}{2304}-2\mathfrak{R}\Li_4(1+i)+\frac{35}{64}\zeta(3)\ln2+\frac{\pi^2}{64}\ln^22\end{eqnarray*}M.H.Zhao,"On logarithmic integrals, harmonic sums and variations"(2020)の36ページによると$$\mathfrak{R}\Li_4(1+i)=-\frac{5}{16}\Li_4\left(\frac{1}{2}\right)+\frac{97\pi^4}{9216}+\frac{\pi^2}{48}\ln^22-\frac{5}{384}\ln^42$$したがって\begin{eqnarray}\mathfrak{R}\sum_{n=1}^\infty\frac{H_n}{n^3}i^n&=&\frac{5}{8}\Li_4\left(\frac{1}{2}\right)-\frac{13\pi^4}{1536}+\frac{35}{64}\zeta(3)\ln2-\frac{5\pi^2}{192}\ln^22+\frac{5\ln^42}{192}\tag{8a}\end{eqnarray}(8)に代入すれば

\begin{eqnarray}\sum_{n=1}^\infty\frac{H_{4n}}{n^3}= 52\Li_4\left(\frac{1}{2}\right)-\frac{43}{80}\pi^4+\frac{91}{2}\zeta(3)\ln2-\frac{13}{6}\pi^2\ln^22+\frac{13}{6}\ln^42\tag{9}\end{eqnarray}

公式の導出③

調和数を含んだ級数(Euler-sum)とゼータ関数 part4

にて得られた$$\sum_{n=1}^\infty\frac{H_n^2}{n^2}=\frac{17}{360}\pi^4$$$$\sum_{n=1}^\infty(-1)^n\frac{H_n^2}{n^2}=2\Li_4\left(\frac{1}{2}\right)-\frac{41\pi^4}{1440}+\frac{7}{4}\zeta(3)\ln2-\frac{\pi^2}{12}\ln^22+\frac{\ln^42}{12}$$および\begin{eqnarray*}\sum_{n=1}^\infty\frac{H_n^2}{n^2}x^n&&=\Li_4(x)-2\Li_4(1-x)+2\ln(1-x)\:\Li_3(1-x)\\&&\quad+\frac{1}{2}\Li_2(x)^2-\ln^2(1-x)\:\Li_2(1-x)-\frac{1}{3}\ln x\ln^3(1-x)+\frac{\pi^4}{45}\end{eqnarray*}に $x=i$ を代入することで、同様の手法により $\sum_{n=1}^\infty\frac{H_{4n}^2}{n^2}$ が求まります。これまでに現れた関係式および$$\Li_2(i)=-\frac{\pi^2}{48}+iG$$を使いましょう。結果として以下を得ます。

\begin{eqnarray}\sum_{n=1}^\infty\frac{H^2_{4n}}{n^2}&=&13\Li_4\left(\frac{1}{2}\right)+\frac{2\pi^4}{15}+\frac{91}{8}\zeta(3)\ln2-\frac{7}{24}\pi^2\ln^22\\&&+\frac{13}{24}\ln^42-4G^2+2\pi G\ln2+4\pi\mathfrak{I}\Li_3(1-i)\tag{10}\end{eqnarray}

$\Li_3(1-i)$ の虚部が分からないのが残念ですが、とりあえずこのように求まります。

おまけ

\begin{eqnarray*}\sum_{n=1}^\infty\frac{(-1)^nH_{2n}}{n^3} &=& 8\sum_{n=1}^\infty\frac{i^{2n}H_{2n}}{(2n)^3}\\ &=&4\left[\sum_{n=1}^\infty\frac{H_n}{n^3}i^n+\sum_{n=1}^\infty\frac{H_n}{n^3}(-i)^n\right]\\ &=&8\mathfrak{R}\sum_{n=1}^\infty\frac{H_n}{n^3}i^n\end{eqnarray*}(8a)より

\begin{eqnarray}\sum_{n=1}^\infty\frac{(-1)^nH_{2n}}{n^3} &=& 5\Li_4\left(\frac{1}{2}\right)-\frac{13\pi^4}{192}+\frac{35}{8}\zeta(3)\ln2-\frac{5\pi^2}{24}\ln^22+\frac{5\ln^42}{24}\tag{11}\end{eqnarray}

おまけ2

(7)より$$\sum_{n=1}^\infty\frac{H_{4n}}{n^2}=\frac{67}{8}\zeta(3)-2\pi G$$ですが、2倍添え字の変形をすると$$\sum_{n=1}^\infty\frac{H_{4n}}{n^2}=2\sum_{n=1}^\infty\frac{H_{2n}}{n^2}+2\sum_{n=1}^\infty\frac{H_{2n}}{n^2}(-1)^n$$右辺第1項は過去記事

調和数を含んだ級数(Euler-sum)とゼータ関数 part7

で得た式$$\sum_{n=1}^\infty\frac{H_{2n}}{n^2}=\frac{11}{4}\zeta(3)$$を用います。すると

\begin{eqnarray}\sum_{n=1}^\infty\frac{(-1)^nH_{2n}}{n^2} &=& \frac{23}{16}\zeta(3)-\pi G\tag{12}\end{eqnarray}

次はこちら:

調和数を含んだ級数(Euler-sum)とゼータ関数 part10

応援のおねがい

Please support me!

まめしば
まめしば

記事を気に入って下さった方、「応援してあげてもいいよ」という方がいらっしゃったら15円から可能なので支援していただければ幸いです。情報発信を継続していくため、サーバー維持費などに充てさせていただきます。

ご支援いただいた方は、こちらで確認できます。

Amazonギフトの場合、
Amazonギフト券- Eメールタイプ – Amazonベーシック
より、金額は空白欄に適当に(15円から)書きこんで下さい。受取人は「mamekebiamazonあっとgmail.com」です(あっとは@に置き換えてください)。贈り主は「匿名」等でOKです。全額がクリエイターに届きます。

OFUSEは登録不要で、100円から寄付できます。金額の90%がクリエイターに届きます。

OFUSEで応援を送る

codocは登録不要で、100円から寄付できます。金額の85%がクリエイターに届きます。

コメントを残す

メールアドレスが公開されることはありません。 が付いている欄は必須項目です

CAPTCHA